Two-Point Boundary Value Problem

Click For Summary
The discussion revolves around solving the two-point boundary value problem defined by the differential equation y'' + ßy = 0 with boundary conditions y'(0)=0 and y'(L)=0. The user successfully solved cases for ß>1 and ß<1 but is struggling with the case when ß=0, where they derived that y''=0 leads to a trivial solution. They express confusion over the textbook's claim that there exists a non-trivial solution y0(x)=1 for ß=0. The conversation highlights the need to carefully consider the implications of boundary conditions on the existence of non-trivial solutions. Ultimately, the user seeks clarification on whether the boundary conditions indeed restrict the solution to trivial outcomes in this specific case.
Jamin2112
Messages
973
Reaction score
12

Homework Statement



y'' + ßy = 0, y'(0)=0, y'(L)=0

Homework Equations



Meh

The Attempt at a Solution



I so already did the ß>1 and ß<1; I'm stuck on the ß=0. It seems easy enough. y'' = 0 -----> y' = A -----> 0=A, 0=A (from the two initial conditions) ------> No non-trivial solution.

But...

The answer in the back of the book says ß0=0, y0(x)=1; ...


??
 
Physics news on Phys.org
Jamin2112 said:

Homework Statement



y'' + ßy = 0, y'(0)=0, y'(L)=0

Homework Equations



Meh

The Attempt at a Solution



I so already did the ß>1 and ß<1; I'm stuck on the ß=0. It seems easy enough. y'' = 0 -----> y' = A -----> 0=A, 0=A (from the two initial conditions) ------> No non-trivial solution.

Starting with y'' = 0 you need to integrate twice to get y, getting two constants.
 
LCKurtz said:
Starting with y'' = 0 you need to integrate twice to get y, getting two constants.

The boundaries it gives me are in terms of y' :wink:


Of course I know
y'' = 0 -----> y' = B ----> y = Ax + b
 
Jamin2112 said:
The boundaries it gives me are in terms of y' :wink:


Of course I know
y'' = 0 -----> y' = B ----> y = Ax + b

But the point is: does your boundary condition force only the trivial solution or can you get a non-trivial solution in this case?
 
Question: A clock's minute hand has length 4 and its hour hand has length 3. What is the distance between the tips at the moment when it is increasing most rapidly?(Putnam Exam Question) Answer: Making assumption that both the hands moves at constant angular velocities, the answer is ## \sqrt{7} .## But don't you think this assumption is somewhat doubtful and wrong?

Similar threads

  • · Replies 1 ·
Replies
1
Views
2K
  • · Replies 2 ·
Replies
2
Views
2K
  • · Replies 4 ·
Replies
4
Views
2K
Replies
2
Views
1K
  • · Replies 8 ·
Replies
8
Views
3K
Replies
4
Views
2K
  • · Replies 6 ·
Replies
6
Views
2K
  • · Replies 2 ·
Replies
2
Views
2K
  • · Replies 1 ·
Replies
1
Views
2K
  • · Replies 8 ·
Replies
8
Views
2K